23
1 AIPTS 2020 (S&T - 17) (E) Answer Key Byju’s Classes: 9873643487 ALL INDIA PRELIMS TEST SERIES - 2020 SCIENCE & TECHNOLOGY + CURRENT AFFAIRS OF JANUARY 2020 ANSWER KEY 1. Ans: (d) Explanation: Global navigation satellite system (GNSS) is a general term describing any satellite constellation that provides positioning, navigation, and timing (PNT) services on a global or regional basis. While GPS is the most prevalent GNSS, below are some of the other significant global and regional navigation systems- Pair 1 is correctly matched: BeiDou, or BDS, is a regional GNSS owned and operated by the People's Republic of China. China is currently expanding the system to provide global coverage with 35 satellites by 2020. BDS was previously called Compass. Pair 2 is incorrectly matched: Galileo is a global GNSS owned and operated by the European Union. The EU declared the start of Galileo Initial Services in 2016 and plans to complete the system of 24+ satellites by 2020. Pair 3 is incorrectly matched: GLONASS (Globalnaya Navigazionnaya Sputnikovaya Sistema, or Global Navigation Satellite System) is a global GNSS owned and operated by Russia. The fully operational system consists of 24+ satellites. Pair 4 is incorrectly matched: Quasi-Zenith Satellite System (QZSS) is a regional GNSS owned by the Government of Japan and operated by QZS System Service Inc. (QSS). QZSS complements GPS to improve coverage in East Asia and Oceania. 2. Ans: (b) Explanation: Betelgeuse, a reddish star that's one of the brightest in the night sky, has been noticeably "fainting," or getting dimmer. The approximately 8.5 million-year- old star, which is part of the Orion constellation, has been one of the most recognizable stars in the sky because of its brightness and coloration. But the recent, dramatic fading has prompted scientists to suggest that the star might be entering a pre- supernova phase, dimming before it collapses and "dies" in a fiery supernova explosion. 3. Ans: (a) Explanation: Option (a) is incorrect: No methane hydrates are found in marginal seas and shelf areas because the pressure at the seafloor is not sufficient to stabilize the hydrates. At the bottom of the expansive ocean basins, scarcely any hydrates are found because there is insufficient organic matter. Methane hydrates occur mainly near the continental margins because enough organic material is deposited there, and the temperature and pressure conditions are favourable. Option (b) is correct: Methane gas is primarily formed by microorganisms that live in the deep sediment layers and slowly convert organic substances to methane. These organic materials are the remains of plankton that lived in the ocean long ago, sank to the ocean floor, and were finally incorporated into the sediments. Option (c) is correct: The total global amount of methane carbon bound up in these hydrate deposits is about 100 to 500 times more carbon than is released annually into the atmosphere by the burning of fossil fuels. At low temperatures the methane hydrates on the seafloor are stable, but if the water and the sea floor become warmer, then the hydrates can break down, because microorganisms then oxidize the resulting methane gas to form the greenhouse gas carbon dioxide (CO2). Methane hydrates have recently become a topic of intense discussion within the context of climate change. Methane, which itself acts as a strong greenhouse gas, does not escape directly out

ALL INDIA PRELIMS TEST SERIES - 2020...2 AIPTS 2020 (S&T - 17) (E) Answer Key Byju’s Classes: 9873643487 of the sea as methane because it is transformed into CO2. But the formation

  • Upload
    others

  • View
    1

  • Download
    0

Embed Size (px)

Citation preview

Page 1: ALL INDIA PRELIMS TEST SERIES - 2020...2 AIPTS 2020 (S&T - 17) (E) Answer Key Byju’s Classes: 9873643487 of the sea as methane because it is transformed into CO2. But the formation

1 AIPTS 2020 (S&T - 17) (E) Answer Key Byju’s Classes: 9873643487

ALL INDIA PRELIMS TEST SERIES - 2020 SCIENCE & TECHNOLOGY +

CURRENT AFFAIRS OF JANUARY 2020

ANSWER KEY

1. Ans: (d) Explanation: Global navigation satellite

system (GNSS) is a general term describing

any satellite constellation that provides

positioning, navigation, and timing (PNT)

services on a global or regional basis. While

GPS is the most prevalent GNSS, below are some of the other significant global and

regional navigation systems-

Pair 1 is correctly matched: BeiDou, or

BDS, is a regional GNSS owned and operated

by the People's Republic of China. China is currently expanding the system to provide

global coverage with 35 satellites by 2020.

BDS was previously called Compass.

Pair 2 is incorrectly matched: Galileo is a

global GNSS owned and operated by the

European Union. The EU declared the start of Galileo Initial Services in 2016 and plans to

complete the system of 24+ satellites by

2020.

Pair 3 is incorrectly matched: GLONASS

(Globalnaya Navigazionnaya Sputnikovaya Sistema, or Global Navigation Satellite

System) is a global GNSS owned and operated

by Russia. The fully operational system

consists of 24+ satellites.

Pair 4 is incorrectly matched: Quasi-Zenith

Satellite System (QZSS) is a regional GNSS owned by the Government of Japan and

operated by QZS System Service Inc. (QSS).

QZSS complements GPS to improve coverage

in East Asia and Oceania.

2. Ans: (b)

Explanation: Betelgeuse, a reddish star

that's one of the brightest in the night sky,

has been noticeably "fainting," or getting

dimmer. The approximately 8.5 million-year-old star, which is part of the Orion

constellation, has been one of the most

recognizable stars in the sky because of its

brightness and coloration. But the recent,

dramatic fading has prompted scientists to

suggest that the star might be entering a pre-supernova phase, dimming before it collapses

and "dies" in a fiery supernova explosion.

3. Ans: (a) Explanation:

Option (a) is incorrect: No methane hydrates

are found in marginal seas and shelf areas

because the pressure at the seafloor is not

sufficient to stabilize the hydrates. At the

bottom of the expansive ocean basins, scarcely any hydrates are found because

there is insufficient organic matter. Methane

hydrates occur mainly near the continental

margins because enough organic material is

deposited there, and the temperature and pressure conditions are favourable.

Option (b) is correct: Methane gas is primarily formed by microorganisms that live

in the deep sediment layers and slowly

convert organic substances to methane.

These organic materials are the remains of

plankton that lived in the ocean long ago, sank to the ocean floor, and were finally

incorporated into the sediments.

Option (c) is correct: The total global

amount of methane carbon bound up in these

hydrate deposits is about 100 to 500 times

more carbon than is released annually into the atmosphere by the burning of fossil fuels.

At low temperatures the methane hydrates on

the seafloor are stable, but if the water and

the sea floor become warmer, then the

hydrates can break down, because microorganisms then oxidize the resulting

methane gas to form the greenhouse gas

carbon dioxide (CO2). Methane hydrates have

recently become a topic of intense discussion

within the context of climate change.

Methane, which itself acts as a strong greenhouse gas, does not escape directly out

Page 2: ALL INDIA PRELIMS TEST SERIES - 2020...2 AIPTS 2020 (S&T - 17) (E) Answer Key Byju’s Classes: 9873643487 of the sea as methane because it is transformed into CO2. But the formation

2 AIPTS 2020 (S&T - 17) (E) Answer Key Byju’s Classes: 9873643487

of the sea as methane because it is transformed into CO2. But the formation and

release of carbon dioxide are considerable

leading to further greenhouse effects.

Option (d) is correct: Methane hydrates are

only stable under pressures in excess of 35 bar and at low temperatures. The seafloor is

thus an ideal location for their formation: the

bottom waters of the oceans and the deep

seabed are almost uniformly cold, with

temperatures from 0 to 4 degrees Celsius. In

addition, below a water depth of about 350 metres, the pressure is sufficient to stabilize

the hydrates.

4. Ans: (c) Explanation: In October 2018, the Supreme

Court had ordered that green crackers would

be allowed to be sold, purchased and used. In

October, 2019, the central government

launched environment-friendly firecrackers.

These green firecrackers are developed by the Council of Scientific and Industrial Research

(CSIR) laboratories. Green crackers don't

contain banned chemicals such as lithium,

arsenic, barium and lead.

'Green' crackers have a small shell size compared to traditional crackers. They are

made with the reduction in the size of shell,

reduced usage of raw materials, dust

suppressants. Green crackers strictly do not

contain elements such as mercury, lithium,

arsenic, and lead, etc. They have permissible sound limits as well as low emission.

Statement 1 is correct: Green crackers are

designed to have 30% less particulate matter

pollution. They release water vapour and

don't allow the dust particles to rise. Statement 2 is incorrect: The Supreme

Court had banned barium nitrate, which is

an important raw material for producing

crackers. Thus, the CSIR-NEERI chose to use

potassium nitrate and zeolite instead.

Statement 3 is correct: Safe Water Releaser (SWAS), Safe Thermite Cracker (STAR) and

Safe Minimal Aluminium (SAFAL) are forms of

the green crackers.

5. Ans: (a) Explanation:

Option (a) is incorrect: Starlink is the name

of a satellite network that the private

spaceflight company SpaceX is developing to

provide low-cost internet to remote locations. It is not being developed by NASA.

Option (b) is correct: The U.S. Federal

Communications Commission (FCC) has

granted SpaceX permission to fly 12,000

satellites and perhaps as many as 30,000

eventually under the Starlink project. SpaceX

launched its first two Starlink test craft, named TinTinA and TinTinB, in 2018. Then,

the first 60 Starlink satellites were launched

on May 23, 2019, aboard a SpaceX Falcon 9

rocket.

Option (c) is correct: Currently satellite internet works using large spacecraft that

orbit 22,236 miles (35,786 km) above a

particular spot on the Earth, which is also

known as high earth or geosynchronous

orbit. However, the fleet of starlink satellites

is operating at lower altitudes of 340 miles (550 kilometers), which comes under low

earth orbit. It is also low enough to pull the

satellites down to Earth by atmospheric drag

in a few years so that they don't become

space junk once they die. Option (d) is correct: Usually to reach far

flung areas internet signals are sent through

physically laid down electric cables. However,

satellite internet works by beaming

information through the vacuum of space,

where it travels 47% faster than in fiber-optic cable. Moreover, from the satellites in high

earth orbit, there are generally significant

time delays in sending and receiving data,

but Starlink satellites being closer to earth

and networking together, will carry large amounts of information rapidly to any point

on Earth, even over the oceans and in

extremely hard-to-reach places. Thus, the

information sent through startlink (Satellite

internet) will travel faster than in fiber-optic

cable.

6. Ans: (b)

Explanation:

Statement 1 is incorrect: Neutrinos are tiny, neutral, elementary particles which

interact with matter via the weak force. These

neutrinos are most abundant in the universe

(they are not rare). The Sun, and all other

stars, produce neutrinos copiously due to

nuclear fusion and decay processes within their core. Neutrino has a very tiny mass. The

first results from the Karlsruhe Tritium

Neutrino (KATRIN) experiment in Germany

reveal that neutrinos weigh at most 1.1

electron volts (eV). Statement 2 is incorrect: Neutrinos are

notoriously difficult to detect in a laboratory

because of their extremely weak interaction

with matter. The background from cosmic

rays and natural radioactivity will make it

almost impossible to detect them on the surface of the Earth. This is the reason most

neutrino observatories are located deep inside

the Earth‘s surface.

Page 3: ALL INDIA PRELIMS TEST SERIES - 2020...2 AIPTS 2020 (S&T - 17) (E) Answer Key Byju’s Classes: 9873643487 of the sea as methane because it is transformed into CO2. But the formation

3 AIPTS 2020 (S&T - 17) (E) Answer Key Byju’s Classes: 9873643487

Statement 3 is correct: One of the earliest laboratories created to detect neutrinos

underground in the world was located more

than 2000 m deep at the Kolar Gold Field

(KGF) mines in India. The first atmospheric

neutrinos were detected at this laboratory in 1965.

7. Ans: (b)

Explanation: The Super-efficient Equipment

and Appliance Deployment (SEAD) of Clean Energy Ministerial (CEM) is a voluntary

collaboration among governments to promote

use of energy efficient appliances worldwide.

At its core, SEAD is about governments

saving energy, turning knowledge into action, and advancing global markets to encompass

energy efficient products. CEM is a high-level

global forum to promote policies and

programs that advance clean energy

technology and to encourage the transition to

a global clean energy economy.

8. Ans: (b)

Explanation: Reflection of sound waves

involves bouncing back of the waves from the

surface of solid or liquid. Whereas, the bending of a sound wave owing to changes in

the wave‘s speed is called refraction.

Statement 1 is incorrect: A sonic boom is a

thunder-like noise a person on the ground

hears when an aircraft or other type of aerospace vehicle flies overhead faster than

the speed of sound, or ―supersonic.‖ Air

reacts like fluid to supersonic objects. As

those objects travel through the air,

molecules are pushed aside with great force

and this forms a shock wave, much like a boat creates a wake in water. This formation

of shock waves, result in Sonic Boom.

Statement 2 is correct: Working of a

stethoscope is based on reflection of sound.

In a stethoscope, the sound of the patient‘s heartbeat reaches the doctor‘s ear by multiple

reflections of sound through a long tube.

Statement 3 is correct: Reflection of sound

is used to measure the distance and speed of

underwater objects. This method is known as

SONAR.

9. Ans: (b)

Explanation:

Statement 1 is incorrect: IRNSS/NavIC is an independent regional navigation satellite

system being developed by India. It is

designed to provide accurate position

information service to users in India as well

as the region extending up to 1500 km from

its boundary, which is its primary service area.

Statement 2 is incorrect: The IRNSS System

is expected to provide a position accuracy of

better than 20 m in the primary service area.

While the GPS is claimed to provide a position accuracy of better than 4 m.

Statement 3 is correct: In NavIC, three

satellites are located in suitable orbital slots

in the geostationary orbit and the remaining

four are located in geosynchronous orbits

with the required inclination and equatorial crossings in two different planes.

10. Ans: (c)

Explanation: Statement 1 is incorrect: The child does not

have an equal proportion of DNA from each

parent. Rather, the majority of the child‘s

DNA is from his parents, with only a small

fraction coming from the mitochondria of the

donor egg or third parent. Statement 2 is correct: When the mother

has damaged mitochondrial DNA, she will

pass this on to all of her children causing

disease of a varied severity depending on the

proportion of healthy and damaged mitochondria the child randomly inherits.

Three parent babies were developed to avoid

this transmission.

Statement 3 is correct: Mitochondria

contains a very small amount of DNA, making

them the only organelle other than the nucleus to house genetic information.

11. Ans: (a)

Explanation:

Statement 1 is correct: DNA replicates on its own, thus it is self-replicating. But RNA

does not replicate on its own. It is synthesized

from DNA when required.

Statement 2 is correct: DNA is located in

the nucleus of a cell and in the mitochondria. Whereas, RNA is found in the cytoplasm,

nucleus, and in the ribosome.

Statement 3 is incorrect: DNA‘s function is

the transmission of genetic information. It

forms as a media for long-term storage.

Whereas , RNA‘s function is the transmission of the genetic code that is necessary for the

protein creation from the nucleus to the

ribosome.

12. Ans: (a) Explanation:

Statement 1 is correct: NetSCoFAN is a

network of research and academic

institutions working in the area of food and

nutrition. The NetSCoFAN would comprise of

Page 4: ALL INDIA PRELIMS TEST SERIES - 2020...2 AIPTS 2020 (S&T - 17) (E) Answer Key Byju’s Classes: 9873643487 of the sea as methane because it is transformed into CO2. But the formation

4 AIPTS 2020 (S&T - 17) (E) Answer Key Byju’s Classes: 9873643487

eight groups of institutions working in different areas viz. biological, chemical,

nutrition & labelling, food of animal origin,

food of plant origin, water & beverages, food

testing, and safer & sustainable packaging.

Statement 2 is incorrect: The Ministry of Health and Family Welfare has launched

NetSCoFAN (Network for Scientific Co-

operation for Food Safety and Applied

Nutrition).

13. Ans: (a) Explanation: Optical mining is an approach

for excavating an asteroid and extracting

water and other volatiles into an inflatable

bag. In Optical Mining, excavating and processing asteroid materials is accomplished

by highly concentrated sunlight which can be

used to drill holes, excavate, disrupt, and

shape an asteroid while the asteroid is

enclosed in a containment bag.

14. Ans: (b)

Explanation:

Statement 1 is incorrect: Augmented

Reality integrates text, graphics, audio and

adds value to the users‘ interaction with the real world while virtual reality replaces the

actual world environment with 3D digital

elements. Augmented Reality combines both

real-world and virtual.

Statement 2 is incorrect: Augmented Reality uses transparent screens for the

projections. But virtual reality uses opaque

screens for the projections. A game like

Pokemon is an application of augmented

reality.

Statement 3 is correct: Augmented Reality combines both real-world and virtual spaces.

Users of AR are still able to sense the real

world around them. This makes the

application more immersive and interactive

among users.

15. Ans: (b)

Explanation:

Statement 1 is correct: Worms infect entire

networks of devices either local or across the internet, by using network interfaces. It uses

each consecutively infected machine to infect

others.

Statement 2 is incorrect: Botnets are

networks of infected computers that are made to work together under the control of an

attacker. They are usually engaged in the

distributed denial of service attacks.

Statement 3 is incorrect: A logic bomb is a

malicious program that uses a trigger to

activate the malicious code. The logic bomb

remains non-functioning until that trigger event happens. Once triggered, a logic bomb

implements a malicious code that causes

harm to a computer. Keylogger records

everything the user types on his/her

computer system to obtain passwords and other sensitive information and send them to

the source of the keylogging program.

16. Ans: (a)

Explanation:

The number of deaths in the US caused due to the mysterious respiratory illness

linked to vaping and e-cigarettes rose to

55, Reuters reported. As of December 27,

the Centers for Disease Control and

Prevention (CDC), a US federal agency, has reported 2,561 cases across the

country who have suffered from the

illness. The CDC is tentatively referring to

the disease as EVALI (e-cigarette, or

vaping, product use associated lung

injury).

E-cigarettes, also called ‗vapes‘ or ‗electronic nicotine delivery systems

(ENDS)‘, are battery-run devices that were

originally marketed as a safer alternative

to smoking traditional cigarettes. In

recent years, there has been a deadly rise

in nicotine addiction in the US, allegedly due to aggressive marketing by

manufacturers.

17. Ans: (d) Explanation:

Statement 1 is correct: Union Commerce

minister has inaugurated the NSE Knowledge

Hub. The ‗NSE Knowledge Hub‘ is an Artificial

Intelligence (AI) powered learning ecosystem.

Statement 2 is correct: The platform has been developed by NSE Academy, a wholly-

owned subsidiary of the National Stock

Exchange (NSE). This platform seeks to assist

the banking, financial, securities and

insurance (BFSI) sector in enhancing skills for their employees. It also aims to help the

academic institutions in preparing future-

ready talent skilled for the financial services

industry.

18. Ans: (a) Explanation: Recently, ISRO discovered that

Geotail is helping Chandrayaan-2 mission in

studying the moon surface.

Statement 1 is correct: Due to interaction of

Earth‘s magnetosphere with Solar wind plasma, Earth‘s magnetosphere is

compressed on the side facing the Sun, and is

stretched into a long tail, on the opposite

Page 5: ALL INDIA PRELIMS TEST SERIES - 2020...2 AIPTS 2020 (S&T - 17) (E) Answer Key Byju’s Classes: 9873643487 of the sea as methane because it is transformed into CO2. But the formation

5 AIPTS 2020 (S&T - 17) (E) Answer Key Byju’s Classes: 9873643487

side. This long tail is known as Geotail. The Geotail is also a region in space that allows

the best observations.

Statement 2 is correct: The charged

particles in the solar wind are embedded in the extended magnetic field of the Sun. Earth

obstructs the solar wind plasma with its

magnetic field. This interaction results in the

formation of a magnetic envelope around

Earth, forming Geotail. Hence, any change in the magnetic field of earth can change the

shape of the ―Geo tail‖.

Statement 3 is incorrect: The region exists

as a result of the interactions between the

Sun and Earth.

19. Ans: (b)

Explanation: SnowEx is a five year program

initiated and funded by NASA Terrestrial

Hydrology Program (THP) to address the most

important gaps in snow remote sensing knowledge. It focuses on airborne campaigns

and field work, and on comparing the various

sensing technologies, from the mature to the

more experimental, in globally-representative

types of snow. The goal is to address the most important gaps in our snow remote sensing

knowledge, and thus lay the groundwork for a

future snow satellite mission.

20. Ans: (b) Explanation:

Statement 1 is incorrect: India successfully

test-fired its first Subsonic cruise missile,

Nirbhay. It is a long range, all-weather,

subsonic cruise missile designed and

developed in India by the Defence Research and Development Organisation (DRDO). The

missile is capable of carrying warheads of up

to 300kg with a speed of 0.6 to 0.7 Mach

(subsonic). Subsonic conditions occur for

Mach numbers less than one, M < 1. Supersonic conditions occur for Mach

numbers greater than one, 1 < M < 3.

Statement 2 is correct: Nirbhay is India's first indigenously designed and developed

long-range state-of-the-art cruise missile,

which can be deployed from multiple

platforms and is capable of carrying

conventional and nuclear warheads. It is a two-stage missile powered by a Solid rocket

motor booster.

Statement 3 is incorrect: Nirbhay has a

range of 800-1000 km and can fly very low to

the ground to avoid detection by enemy radar

called terrain hugging capability. The Nirbhay cruise missile is an Indian version of the

American Tomahawk.

21. Ans: (c) Explanation: Solar Orbiter is a mission

dedicated to solar and heliospheric physics.

Statement 1 is incorrect: The mission is a

joint collaboration between NASA and the

European Space Agency.

Statement 2 is correct: This is the first mission that will provide images of the sun's

north and south poles using a suite of six

instruments on board that will capture the

spacecraft's view. Having a visual

understanding of the sun's poles is important because it can provide more insight about the

sun's powerful magnetic field and how it

affects Earth.

Statement 3 is correct: The mission will

work in tandem with NASA's Parker Solar

Probe, which is currently orbiting the sun on a seven-year mission and just completed its

fourth close approach of the star. It was

launched in August 2018 and will eventually

come within four million miles of the sun --

the closest a spacecraft has ever flown by our star.

22. Ans: (a)

Explanation: NEONs are computationally

created virtual humans — the word derives from NEO (new) + humaN. The first project

of Star Labs, NEONs, is being called the

world‘s first artificial humans. They look and

behave like real humans, and could one day

develop memories and emotions — though

from behind a 4K display. It is a humanoid AI chatbot. It aims to deliver a personalised

experience with an AI. It has the ability to

show emotions and intelligence. It is capable

of conversing and sympathising like real

human beings. The AI also understands Hindi, Spanish and other languages. Under

the hood, Neon runs on a proprietary

technology platform, Core R3, where R3

stands for Reality, Realtime, and

Responsiveness. It is the brainchild of Pranav

Mistry, President, and CEO of Star Labs.

Page 6: ALL INDIA PRELIMS TEST SERIES - 2020...2 AIPTS 2020 (S&T - 17) (E) Answer Key Byju’s Classes: 9873643487 of the sea as methane because it is transformed into CO2. But the formation

6 AIPTS 2020 (S&T - 17) (E) Answer Key Byju’s Classes: 9873643487

Neon aims to deliver a deeply personalised experience with an AI.

23. Ans: (d)

Explanation: The Aditya-L1 project will enable a comprehensive understanding of the

dynamical processes of the sun and address

some of the outstanding problems in solar

physics.

Statement 1 is correct: It will be ISRO‘s

first mission to the Sun. Statement 2 is correct: The Indian Space

Research Organisation (ISRO) is preparing to

send its first scientific expedition to study the

Sun. Named Aditya-L1, the mission, expected

to be launched early next year, will observe the Sun from a close distance, and try to

obtain information about its atmosphere and

magnetic field.

Statement 3 is correct: L1 refers to

Lagrangian/Lagrange Point 1, one of five

points in the orbital plane of the Earth-Sun system. Lagrange Points, named after Italian-

French mathematician Joseph-Louis

Lagrange, are positions in space where the

gravitational forces of a two-body system (like

the Sun and the Earth) produce enhanced regions of attraction and repulsion. These can

be used by spacecraft to reduce fuel

consumption needed to remain in position. A

Satellite placed in the halo orbit around the

Lagrangian point 1 (L1) of the Sun-Earth

system has the major advantage of continuously viewing the Sun without any

occultation/ eclipses.

24. Ans: (b)

Explanation: Recently, messaging platform WhatsApp‘s revealed that Indian journalists

and human rights activists were among

some 1,400 people globally spied upon using

surveillance technology developed by Israel-

based NSO Group. In this case, a malicious code, named Pegasus, exploited a bug in the

call function of WhatsApp to make its way

into the phones of those select users, where it

would potentially have had access to every bit

of information.

The code is transmitted by calling the target phone on WhatsApp. The code enters the

phone even if the call is not answered. Once

installed, Pegasus can send the target‘s

contacts, calendar events, phone calls on and

messages on communication apps like WhatsApp and Telegram to the spyware‘s

controller. It could steal messages from even

services that offer encryption because it was

taking the messages before the encryption

process, according to anti-malware service

Kaspersky. The controller can also turn the

phone into a spying device by switching on its camera or microphone.

25. Ans: (c)

Explanation: Indian Space Research Organisation (ISRO) recently launched the

Polarimetry Doppler Weather Radar at Satish

Dhawan Space Centre, Sriharikota.

Statement 1 is correct: The Polarimetry

Doppler Weather Radar provides advanced

information, enhances the lead-time essential for saving lives and property in the event of a

natural disaster associated with severe

weather.

Statement 2 is correct: The severity of the

weather systems can be quantitatively estimated more accurately and more precise

advanced warnings can be generated for

saving human lives and property. While

conventional radars are able to track and

predict cyclones, the Doppler Weather Radar

provides detailed information on a storm‗s internal wind flow and structure.

26. Ans: (d)

Explanation: India is playing an enormous

interest in establishing a relationship with various countries around the globe.

Integration of the military exercises is an

unavoidable obligation for the contemporary

combat zone. The joint military is essential

for the cooperation of countries apart from economic cooperation and the prevailing hard

situation. The most significant benefit of joint

military exercises is ‗strategic signaling‘

Pair 1 is correctly matched: SURYA KIRAN

with Nepal is an important exercise of India

in terms of the security challenges faced by both the nations in the realm of changing

facets of global terrorism.

Pair 2 is correctly matched: The eighth

edition of the joint India-China military

training exercise Hand-in-Hand 2019 was conducted in Meghalaya.

Pair 3 is correctly matched: The joint

military exercise between India and

Seychelles is named 'Lamitye' which means

friendship in Creole (local dialect of

Seychelles). Pair 4 is correctly matched: Ninth edition of

India-Bangladesh joint military exercise

Sampriti, was commenced at Umroi,

Meghalaya, February 2020.

27. Ans: (a)

Explanation:

Option (a) is correct: Recently, EMISAT

satellite was launched by the Indian Space

Research Organisation (ISRO) to enhance

Page 7: ALL INDIA PRELIMS TEST SERIES - 2020...2 AIPTS 2020 (S&T - 17) (E) Answer Key Byju’s Classes: 9873643487 of the sea as methane because it is transformed into CO2. But the formation

7 AIPTS 2020 (S&T - 17) (E) Answer Key Byju’s Classes: 9873643487

India's surgical warfare capacity. EMISAT has been developed under DRDO's Project

Kautilya which aims to boost India's space

surveillance capacity. The project is named

after the ancient Indian economist who

emphasised the importance of spying for a king to protect his kingdom.

Option (b) is incorrect: Project Sashakt aims

to address the bad loans problem in the

financial sector of India's economy.

Option (c) is incorrect: Project Insights is

related to monitoring high value transactions and detecting tax evasions.

Option (d) is incorrect: Project Mausam is

related to foreign policy initiative to

reestablish India‘s ties with its ancient trade

partners.

28. Ans: (a)

Statements 1 is correct: An electric motor is

highly efficient in converting energy stored in

the battery to making the car move: it is typically 60-80 percent efficient. The

efficiency of an internal combustion engine in

converting the energy stored in gasoline to

making the car move is typically 15 percent.

Statement 2 is incorrect: Gasoline is about 100 times more energy dense than a lithium

ion battery. However, this difference in

energy density is partially mitigated by the

very high efficiency of an electric motor in

converting energy stored in the battery to

making the car move.

29. Ans: (a)

Explanation:

Statement 1 is correct: The government in

its budget 2020 has announced a National Mission on Quantum Technologies &

Applications (NM-QTA) with a total budget

outlay of Rs 8000 Crore for a period of five

years to be implemented by the Department

of Science & Technology (DST). Statement 2 is incorrect: Quantum

Artificial Intelligence Laboratory (QuAIL) is

NASA‘s initiative. It aims to demonstrate that

quantum computing and quantum algorithms

may someday dramatically improve the

agency‘s ability to address difficult optimization and machine learning problems

arising in NASA's aeronautics, Earth and

space sciences, and space exploration

missions.

DST launched Quantum Information Science and Technology (QuEST), a programme

wherein the government will invest ₹80 crore

in the next three years to fund research

directed to build quantum computers,

channels for quantum communication and

cryptography, among other things.

30. Ans: (a) Explanation:

Statement 1 is correct: General Conference

on Weights and Measures (CGPM) is the

highest international body of the world for accurate and precise measurements. The

International Bureau of Weights and

Measures (BIPM), the main executive body of

CGPM, which has the responsibility of

defining the International System of Units

(SI). Statement 2 is incorrect: The new definition

of kg involves accurate weighing machines

called 'Kibble balance', which uses Planck's

Constant to measure the mass of an object

using a precisely measured electromagnetic force. Kilograms were earlier defined by the

weight of a platinum-based ingot called ―Le

Grand K‖, which had been at the forefront of

the international system of measuring

weights since 1889.

31. Ans: (a)

Explanation:

Statement 1 is correct: The IAU serves as

the internationally recognized authority for

assigning designations to celestial bodies and surface features on them. India is one of the

members of this organisation. Its mission is

to promote and safeguard the science of

astronomy in all its aspects, including

research, communication, education and development, through international

cooperation.

Statement 2 is incorrect: The International

Astronomical Union (IAU) was founded in

1919. In 2019, it celebrated its 100th

anniversary. To commemorate this milestone, the IAU is organising a year-long celebration

to increase awareness of a century of

astronomical discoveries as well as to support

and improve the use of astronomy as a tool

for education, development and diplomacy under the central theme "Under One Sky". On

the other hand, 2009 was declared as the

International Year of Astronomy. It was a

year-long celebration of astronomy that took

place in 2009 to coincide with the 400th

anniversary of the first recorded astronomical observations with a telescope by Galileo

Galilei and the publication of Johannes

Kepler's Astronomia nova in the 17th

century.

32. Ans: (c)

Explanation: Blockchain technology is most

simply defined as a decentralized, distributed

ledger that records the provenance of a digital

asset. When a block stores new data it is

Page 8: ALL INDIA PRELIMS TEST SERIES - 2020...2 AIPTS 2020 (S&T - 17) (E) Answer Key Byju’s Classes: 9873643487 of the sea as methane because it is transformed into CO2. But the formation

8 AIPTS 2020 (S&T - 17) (E) Answer Key Byju’s Classes: 9873643487

added to the blockchain. Blockchain, as its name suggests, consists of multiple blocks

strung together. In order for a block to be

added to the blockchain, however, four things

must happen:

1. A transaction must occur 2. That transaction must be verified

3. That transaction must be stored in a

block. After your transaction has been

verified as accurate, it gets the green light

4. That block must be given a unique,

identifying code called hash

Statement 1 is correct: Distributed Ledger

Technology refers to a novel and fast-evolving

approach to recording and sharing data

across multiple data stores (or ledgers). This technology allows for transactions and data

to be recorded, shared, and synchronized

across a distributed network of different

network participants. Blockchain, is a type of

Distributed Ledger Technology (DLT), which

makes the history of any digital asset unalterable and transparent through the use

of decentralization and cryptographic

hashing.

Statement 2 is correct: Blockchain enables

direct transfers of digital value or tokens between two counterparties and decentralized

record-keeping, removing the need for an

intermediary or central authority who

controls the ledger.

33. Ans: (a) Explanation: DME (CH3OCH3) is the

simplest ether compound that can be

produced from methanol or directly from

syngas. DME(Dimethyl Ether) is a product of catalytic dehydration of methanol, which can

be blended with LPG and can be an excellent

substitute for diesel in Large buses and

trucks. To adopt Methanol as a transport

fuel, it requires minimal infrastructure

modifications and capital both in vehicles and in terminal and distribution infrastructure.

Additional Information

CNG (Compressed Natural Gas) is a mixture of hydrocarbons consisting of

approximately 80 to 90 percent methane

in gaseous form. It is an odorless, tasteless gas that is naturally created by

the decomposition of organic matter over

time. When CNG fuel combusts,its

byproduct is CO2 and H2O or carbon

dioxide and water.

Producer gas is fuel gas that is manufactured from material such as coal,

as opposed to natural gas. A combustible

mixture of nitrogen, carbon monoxide, and hydrogen, generated by passing air

with steam over burning coke or coal in a

furnace and used as fuel. It is also called

air gas.

DME (Dimethyl Ethanol) is not a gaseous version of Ethanol, it is a gaseous version of Methanol. Ethanol is a clear, colorless

liquid which is widely used as a solvent

and preservative in pharmaceutical

preparations as well as serving as the

primary ingredient in alcoholic beverages.

34. Ans: (a)

Explanation: U.S.A launched project Blue

Dot, the multi-stakeholder initiative to set up

high quality infrastructure pegged to global standards, which already has the support of

Japan and Australia. The Blue Dot Network

(BDN), which came into being in November at

the Indo-Pacific Business Forum in Bangkok,

seeks to set up a globally recognised

certification system for infrastructure projects to boost transparency and sustainability. The

BDN would reportedly bring together

governments, the private sector and civil

society to gainfully promote roads, ports,

bridges etc, with a special emphasis on the Indo-Pacific region.

35. Ans: (a)

Explanation:

Statement 1 is correct: A fuel cell is a device that converts chemical potential energy into

electrical energy. It supplies electrical energy

in the form of direct current. The products of

the reaction in the cell are water, electricity,

and heat.

Statement 2 is incorrect: The hydrogen fuel cells do not store solid oxidisers. Oxygen is

taken from the air and hydrogen is generally

made from the decomposition of a

hydrocarbon source. Oxygen reacts with

hydrogen to form water. This is a redox process in which electrons are transferred,

the trick of a fuel cell is to use the electrons

generated during the reaction to perform

work.

Statement 3 is incorrect: Fuel cells are

often referred to as continuously operating batteries. They exploit electrolysis reactions

in a similar manner to traditional batteries

however the reagents are constantly

resupplied to the cell and hence they do not

become discharged like traditional batteries.

36. Ans: (b)

Explanation: The World Health Organization

has endorsed TrueNat, an indigenous

Page 9: ALL INDIA PRELIMS TEST SERIES - 2020...2 AIPTS 2020 (S&T - 17) (E) Answer Key Byju’s Classes: 9873643487 of the sea as methane because it is transformed into CO2. But the formation

9 AIPTS 2020 (S&T - 17) (E) Answer Key Byju’s Classes: 9873643487

molecular diagnostic tool for tuberculosis (TB) diagnosis. The disease remains a threat to

public health and is the top infectious cause

of death globally. The Truenat TB test can

diagnose TB in one hour as well as testing for

resistance to the drug rifampicin. The test has been developed by the Indian firm MolBio

Diagnostics. The test works by rapid

detection of TB bacteria using the polymerase

chain reaction (PCR) technique.

37. Ans: (a) Explanation:

Pair 1 is correctly matched: Bunchy top of

banana disease is caused by the banana

bunchy top virus (BBTV). Initially, dark green streaks appear in the veins of the lower

portion of the leaf midrib and the leaf stem of

banana plants. They appear to be ―bunched‖

at the top of the plant, the symptom for which

this disease is named. Severely infected

banana plants will not fruit usually. Pair 2 is incorrectly matched: Panama

disease is a wilting disease caused by the

fungus Fusarium oxysporum. It kills plants

by clogging their vascular system. In a long-

feared development, an extremely damaging banana disease has apparently reached Latin

America. Already widespread in Asia, the

disease can wipe out entire plantations.

Pair 3 is incorrectly matched: Blight of rice

disease is caused by the Bacteria

Xanthomonas oryzae. The disease is most likely to develop in areas that have weeds and

stubbles of infected plants. It can occur in

both tropical and temperate environments,

particularly in irrigated and rainfed lowland

areas.

38. Ans: (c)

Explanation:

Option (a) is incorrect: The Doomsday Clock

is a symbol that represents the likelihood of a man-made global catastrophe.

Option (b) is incorrect: The WHO Strategic

Health Operations Centre (SHOC) monitors

global public health events around the clock,

and facilitates international collaboration

during public health emergencies and daily operations. Epigenetic clock is not related to

Monitoring global public health events.

Option (c) is correct: A small clinical study

in California has suggested for the first time

that it might be possible to reverse the body‘s epigenetic clock, which measures a person‘s

biological age. The epigenetic clock relies on

the body‘s epigenome, which comprises

chemical modifications, such as methyl

groups, that tag DNA. The pattern of these

tags changes during the course of life, and

tracks a person‘s biological age, which can lag behind or exceed chronological age.

Option (d) is incorrect: Genetic clock is

referred to the technique of calculation of

mutation rate, which is used for calculation

of prehistoric life forms. By calculating the rate of mutation, we may conclude the history

of divergence of life forms from a point. In this

technique, the sequences of nucleotides,

amino acids etc., are calculated.

39. Ans: (c) Explanation:

Statement 1 is correct: The belched out C02

(greenhouse gas emission) both from using

Methanol and while producing Methanol can be tapped back to produce Methanol. Thus, a

seamless loop of CO2 sequestration cycle is

created to perpetually burn fuels without

polluting the environment at all. C02 from

steel plants, Thermal Power plants, Cement

Plants etc. can be tapped in large quantities to produce Methanol.

Statement 2 is correct: Methanol, although

slightly lower in energy content than petrol

and diesel, can replace both petrol and diesel

in transport sector (road, rail and marine), energy sector (comprising of DG sets, boilers,

process heating modules, tractors and

commercial vehicles) and retail cooking

replacing LPG (partially), Kerosene and wood

charcoal. Blending of 15% methanol in

gasoline can result in at least 15% reduction in import of gasoline/ crude.

Statement 3 is correct: Methanol burns

efficiently in all internal combustion engines,

produces no particulate matter, no soot,

almost nil SOX and NOX emissions (NEAR ZERO POLLUTION).

Statement 4 is incorrect: Methanol is a

scalable and sustainable fuel that can be

produced from a variety of feedstocks like

Natural Gas, Coal (Indian High Ash Coal),

Biomass, Municipal Solid waste and most importantly from CO2.

40. Ans: (d)

Explanation: As the light emitted by distant

galaxies passes by massive objects in the universe, the gravitational pull from these

objects can distort or bend the light. This is

called gravitational lensing. A gravitational

lens can occur when a huge amount of

matter, like a cluster of galaxies, creates a gravitational field that distorts and magnifies

the light from distant galaxies that are behind

it but in the same line of sight. The effect is

like looking through a giant magnifying glass.

It allows researchers to study the details of

Page 10: ALL INDIA PRELIMS TEST SERIES - 2020...2 AIPTS 2020 (S&T - 17) (E) Answer Key Byju’s Classes: 9873643487 of the sea as methane because it is transformed into CO2. But the formation

10 AIPTS 2020 (S&T - 17) (E) Answer Key Byju’s Classes: 9873643487

early galaxies too far away to be seen with current technology and telescopes.

Applications of Gravitational Lensing are-

Study the galaxies, which are very far away

Observe invisible things in the Universe like dark matter

Understand formation of stars

Understand the past Universe

41. Ans: (d)

Explanation:

Option (a) is incorrect: Maanav is India‘s

first 3D-printed humanoid robot Weighing 2kg, Manav which means man in Sanskrit

has in-built vision and sound processing

capability, which will allow it to talk and act

exactly like a human. It is not part of the

Gaganyaan mission.

Option (b) incorrect: Ai-da is the world‘s first ultra-realistic AI robot artist. She can draw,

and is a performance artist. It is not part of

the Gaganyaan mission.

Option (c) is incorrect: FEDOR or Final

Experimental Demonstration Object Research, was built to assist space station

astronauts. It is built by Russia. It is not part

of the Gaganyaan mission.

Option (d) is correct: Recently, Indian Space

Research Organisation (ISRO) unveiled its

first ‗woman‘ astronaut, named Vyom Mitra who will ride to space in the first test flight of

the human space mission, Gaganyaan. It is a

half-humanoid with no legs and capable of

switching panel operations, performing

Environment Control and Life Support Systems (ECLSS) functions, conversations

with the astronauts, recognising them and

solving their queries. The humanoid has been

developed by the ISRO Inertial Systems Unit,

Thiruvananthapuram.

42. Ans: (d)

Explanation:

Pair 1 is correctly matched: Heavy Water is

used as moderator in the nuclear reactors.

Heavy Water(D2O) or deuterium oxide is made up of two atoms of deuterium and one

atom of oxygen. Moderator is required in a

Nuclear reactor to slow down the neutrons

produced during the fission reaction so that

the chain reaction can be sustained. Heavy Water is an excellent moderator due to its

high moderating ratio and low absorption

cross section for neutrons.

Pair 2 is correctly matched: Hydrogen

peroxide is used in water pollution control

treatment. For example Hydrogen peroxide is

used for pollution control treatment of domestic and industrial effluents, oxidation of

cyanides, restoration of aerobic conditions to

sewage wastes, etc. Nowadays it is also used

in Environmental Chemistry.

Pair 3 is correctly matched: Washing soda is used in removal of hardness of water.

Permanent hardness is due to the presence of

soluble salts of magnesium and calcium in

the form of chlorides and sulphates in water.

43. Ans: (a) Explanation: Indian Railways has

commissioned the country's first government

Waste to Energy Plant, having capacity of 500

Kg waste per day, in Mancheswar Carriage Repair Workshop at Bhubaneswar in East

Coast Railway. This Waste to Energy Plant

has been constructed in three months. The

plant installed converts waste collected from

the garbage disposal units into carbon

powder, diesel and gas using Polycrack Technology.

This Waste to Energy Plant, a patented

technology called POLYCRACK, is first-of-its-

kind in Indian Railways and fourth in India.

It is the world's first patented heterogeneous catalytic process which converts multiple feed

stocks into hydrocarbon liquid fuels, gas,

carbon and water. Polycrack Plant can be fed

with all types of Plastic, Petroleum sludge,

Un-segregated MSW (Municipal Solid Waste)

with moisture up to 50%, E–Waste, Automobile fluff, Organic waste including

bamboo, garden waste etc., and Jatropha

fruit and palm bunch. Waste generated from

Mancheswar Carriage Repair Workshop,

Coaching Depot and Bhubaneswar Railway Station will be feeder material for this plant.

44. Ans: (a)

Explanation: ASTRA is a Beyond Visual

Range (BVR) class of Air-to-Air Missile (AAM) system designed to be mounted on fighter

aircraft. The missile is designed to engage

and destroy highly manoeuvring supersonic

aircraft. The missile has all weather day and

night capability. The missile is being

developed in multiple variants to meet specific requirements. The ASTRA Mk-I

Weapon System integrated with SU-30 Mk-I

aircraft is being inducted into the Indian Air

Force (IAF). It has been Indigenously designed

by DRDO.

45. Ans: (b)

Explanation:

Statement 1 is incorrect: India‘s first Super

Fab Lab was inaugurated in Kerala. The

Page 11: ALL INDIA PRELIMS TEST SERIES - 2020...2 AIPTS 2020 (S&T - 17) (E) Answer Key Byju’s Classes: 9873643487 of the sea as methane because it is transformed into CO2. But the formation

11 AIPTS 2020 (S&T - 17) (E) Answer Key Byju’s Classes: 9873643487

country's first Super Fab Lab, which will give a major push to the hardware industry in the

country and was launched at the Integrated

Startup Complex of the Kerala Startup

Mission (KSUM). The Super Fab Lab will

function in collaboration with the Massachusetts Institute of Technology (MIT).

Statement 2 is correct: Fab Labs are

fabrication laboratories that offer digital

fabrication and computation. The Fab Labs

programme was set up by the state

government to encourage startups in printed electronics and related fields.

46. Ans: (a)

Explanation: Statement 1 is correct: ISRO has initiated

‗Project NETRA‘ – an early warning system in

space to detect debris and other hazards to

Indian satellites. Under the project, the ISRO

plans to put up many observational facilities:

connected radars, telescopes; data processing units and a control centre. They can, among

others, spot, track and catalogue objects as

small as 10 cm, up to a range of 3,400 km

and equal to a space orbit of around 2,000

km. Statement 2 is incorrect: Project Netra is

totally indigenous. The project will give India

its own capability in space situational

awareness (SSA) like the other space powers

— which is used to ‗predict‘ threats from

debris to Indian satellites.

47. Ans: (c)

Explanation:

Pair 1 is incorrectly matched: The

European Space Agency (ESA) has approved the budget of Hera, the European component

of the mission to slam a spacecraft into an

asteroid. Hera is the European contribution

to an international double-spacecraft

collaboration. The project aims to study the effectiveness of an impact to ward off an

impending asteroid threat.

Pair 2 is correctly matched: European

Space Agency (ESA) and the Japan Aerospace

Exploration Agency (JAXA) successfully sent

two probes on a joint mission to Mercury. European and Japanese space agencies said

an Ariane 5 rocket successfully lifted a

spacecraft carrying two probes into orbit for a

joint mission to Mercury, the closest planet to

the sun. When it arrives, BepiColombo will release two probes — Bepi and Mio — that

will independently investigate the surface and

magnetic field of Mercury.

Pair 3 is correctly matched: InSight is part

of NASA‘s Discovery Program, managed by

the agency‗s Marshall Space Flight Center in

Huntsville, Alabama. It will be the first mission to peer deep beneath the Martian

surface, studying the planet‗s interior by

measuring its heat output and listening for

marsquakes, which are seismic events similar

to earthquakes on Earth.

48. Ans: (a)

Explanation:

Option (a) is correct: India successfully test-

fired the 3,500 km strike range nuclear-capable K-4 submarine-launched ballistic

missile (SLBM) off the coast of Andhra

Pradesh. The missile is one of the two

underwater missiles that are being developed

by the Defence Research and Development Organisation (DRDO). The other underwater

missile being developed is K5, a 5,000 km

range SLBM.

Option (b) is incorrect: Iron Dome is Israel's

anti missile system.

Option (c) is incorrect: Khordad 15 is Iran‘s air defense system.

Option (d) is incorrect: Saudi Arabia boasts

an arsenal of sophisticated and expensive air

defense equipment. They have the American-

made Patriot missile defense system, German-made Skyguard air defense cannons

and France‘s Shahine mobile anti-aircraft

system.

49. Ans: (a) Explanation:

Statement 1 is incorrect: Meteors do

not revolve around the Sun in an elliptical

orbit. A meteor is usually a small object that

occasionally enters the earth‘s atmosphere.

At that time it has a very high speed. The friction due to the atmosphere heats it up. It

glows and evaporates quickly.

Statement 2 is correct: There is a large gap

in between the orbits of Mars and Jupiter.

This gap is occupied by a large number of small objects that revolve around the Sun.

These are called asteroids.

Statement 3 is incorrect: The Sun and the

celestial bodies which revolve around it form

the solar system. It consists of a large

number of bodies such as planets, comets, asteroids and meteors. A Constellation does

not form part of the Solar system.

50. Ans: (c) Explanation: The Outer Space Treaty, to

which India and most nations are party,

stipulates that outer space and celestial

bodies must be used ―exclusively for peaceful

purposes,‖ and prohibits testing of weapons

and conduct of military manoeuvres

Page 12: ALL INDIA PRELIMS TEST SERIES - 2020...2 AIPTS 2020 (S&T - 17) (E) Answer Key Byju’s Classes: 9873643487 of the sea as methane because it is transformed into CO2. But the formation

12 AIPTS 2020 (S&T - 17) (E) Answer Key Byju’s Classes: 9873643487

Statement 1 is correct: The Outer Space Treaty prohibits only weapons of mass

destruction in outer space, not ordinary

weapons.

Statement 2 is correct: India is a signatory

to this treaty, and ratified it in 1982. Statement 3 is incorrect: India is not in

violation of any international law or Treaty.

ASAT weapons are not expressly included

within the purview of the Outer Space Treaty

which, under Article IV, only bans states from

―placing in orbit around the Earth any objects carrying nuclear weapons or any other kinds

of weapons of mass destruction, installing

such weapons on celestial bodies, or

stationing such weapons in outer space in

any other manner.‖ This provision would not apply to ASAT weapons, since they only travel

through outer space and are not fixed in a

particular position. Additionally, they are not

nuclear weapons, or weapons of mass

destruction.

51. Ans: (b)

Explanation:

Statement 1 is correct: Amoebic Dysentery

is caused by protozoan parasite Entamoeba histolytica.

Statement 2 is correct: Malaria is a

protozoan infection of the red blood cells,

transmitted by the bite of anopheles

mosquito. Malaria is caused by the protozoa

of the genus Plasmodium. Statement 3 is incorrect: Typhoid fever is

caused by Salmonella typhi bacteria.

Statement 4 and 5 are incorrect:

Chickenpox is a contagious disease caused by

the varicella-zoster virus and Hepatitis A is a liver disease caused by the hepatitis A virus

(HAV).

52. Ans: (c)

Explanation: Reusable Launch Vehicle – Technology Demonstrator (RLV-TD) is one of

the most technologically challenging

endeavors of ISRO towards developing

essential technologies for a fully reusable

launch vehicle to enable low cost access to

space. The configuration of RLV-TD is similar to that of an aircraft and combines the

complexity of both launch vehicles and

aircraft. The winged RLV-TD has been

configured to act as a flying test bed to

evaluate various technologies, namely, hypersonic flight, autonomous landing and

powered cruise flight. In future, this vehicle

will be scaled up to become the first stage of

India‘s reusable two stage orbital launch

vehicle.

53. Ans: (d) Explanation:

Statement 1 is correct: The Indian remote

sensing satellite (IRS) programme which was

started in 1988 with the launch of IRS-1A, is a major step forward in the overall

programme for using space technology for

defined applications in India. The Cartosat

series of satellites has been the flag-bearer of

the Indian Remote Sensing (IRS) system.

Statement 2 is correct: The Cartosat satellites are a series of Indian optical earth

observation satellites. They are built and

operated by ISRO. They are used for Earth‘s

resource management, defence services and

monitoring. The Department of Space (DoS) had launched and managed the IRS series of

remote sensing satellites. Considering

increased demand for large scale and

topographic mapping data, the DoS launched

the expanded Cartosat series.

Statement 3 is correct: ISRO launched CARTOSAT 3, which is the latest in this

series. History of Cartosat series:

CARTOSAT–1: It is the first Indian Remote Sensing Satellite capable of providing in-

orbit stereo images. It was launched in

May 2005 by PSLV- C6. The images were used for Cartographic applications

meeting the global requirements. Cameras

of this satellite had a resolution of 2.5m.

CARTOSAT–2: It is an advanced remote sensing satellite with a single

panchromatic camera (PAN) capable of

providing scene-specific spot imageries for cartographic applications. The camera is

designed to provide imagery with better

than one meter spatial resolution.

CARTOSAT-3: ISRO launched advanced earth imaging and mapping satellite

CARTOSAT-3 along with 13 other

commercial nano-satellites for the US using PSLV-C47 on November 27, 2019. It

is the most advanced earth observation

satellite built by ISRO. CARTOSAT-3 is

the ninth in the series, which is a third-

generation agile satellite with high-resolution imaging capabilities. It has an

overall mass of over 1,600 kilograms with

a mission life of about five years.

Cartosat-3 will address the increased

user‘s demands for: large scale urban

planning, rural resource and infrastructure development, coastal land

use and land cover, cartography (map-

making applications), road-network

monitoring, changing detection in

bringing out geographical and man-made features.

Page 13: ALL INDIA PRELIMS TEST SERIES - 2020...2 AIPTS 2020 (S&T - 17) (E) Answer Key Byju’s Classes: 9873643487 of the sea as methane because it is transformed into CO2. But the formation

13 AIPTS 2020 (S&T - 17) (E) Answer Key Byju’s Classes: 9873643487

54. Ans: (a) Explanation: Nuclear Fuel Complex under

Department of Atomic Energy has developed

a material to be used in ISRO planned moon

mission called Inconel 718. Option (a) is correct: It is considered as a

refractory super-alloy since it can be used

permanently above 600°C. The alloy

associates good creep and rupture strength

with a high resistance to fatigue.

Option (b) is incorrect: It is not a cryogenic fuel with very low density.

Option (c) is incorrect: Inconel 718

accounts for up to 50% of the weight of

aircraft turbojet engines, being the main

component for discs, blades and casing of the high pressure section of the compressor and

discs as well as some blades of the turbine

section. It also finds several applications in

rocket engines and cryogenic environments

due to its good toughness at low temperature

(preserving parts from a brittle fracture). It is not used for space suits of astronauts.

Option (d) is incorrect: It possesses long

time strength and toughness at higher

temperature along with confinement of

corrosion resistance up to high temperature. This alloy has excellent weldability when

compared to the nickel-base superalloys

hardened by aluminum and titanium.

55. Ans: (b) Explanation:

Statement 1 is correct: Blood is a special

connective tissue consisting of a fluid matrix,

plasma, and formed elements.

Statement 2 is incorrect: Erythrocytes (Red

blood cells) are the most abundant of all the cells in blood. A healthy adult man has, on

average, 5 million to 5.5 million of RBCs mm3

of blood. RBCs are formed in the red bone

marrow in adults.

Statement 3 is correct: Leukocytes are also known as white blood cells (WBC) as they are

colourless due to the lack of haemoglobin.

They are nucleated and are relatively lesser in

number which averages 6000-8000 mm3 of

blood. Leukocytes are generally short lived.

56. Ans: (c)

Explanation:

Option (a) is incorrect: Kerala Floods:

‗‗Operation Madad and Sahyog‘: were rescue operations conducted by Navy and Army

during Kerala Floods

Option (b) is incorrect: Yemen Evacuation:

Operation Raahat was an operation of the

Indian Armed Forces to evacuate Indian

citizens and foreign nationals from Yemen.

Option (c) is correct: The code name given to Balakot Air Strike was Operation Bandar.

Option (d) is incorrect: Nepal Earthquake:

Operation Maitri (Operation Amity) was a

rescue and relief operation in Nepal by the

government of India and Indian armed forces in the aftermath of the April 2015 Nepal

earthquake.

57. Ans: (b)

Explanation: Option (a) is incorrect: Carbon nanotubes

are cylindrical carbon molecules with the

properties that make them potentially useful

in a variety of applications in nanotechnology,

electronics, optics and other fields of materials science. The Vikram Sarabhai

Space Centre has not developed indigenous

technology to manufacture this type of high-

power batteries for automobiles and e-

vehicles.

Option (b) is correct: The government has asked Indian Space Research Organisation

(ISRO) to allow manufacturers interested in

producing indigenous lithium-ion batteries.

This includes those from the private sector, to

obtain the technology for its mass production. The Vikram Sarabhai Space Centre under

ISRO has developed indigenous technology to

manufacture such high-power batteries for

automobiles and e-vehicles and their

feasibility tests in vehicles have been

successful. Over half a dozen major automobile companies, battery

manufacturers and public sector

undertakings have already approached ISRO.

At present, these batteries are imported in

India. These batteries have high-power, they weigh less and their volume is much less as

well in comparison to conventional batteries.

Option (c) is incorrect: Hydrogen powered

Tata Starbus fuel cell bus is a zero-emission

vehicle best suited for inter-city

transportation for the masses and has been developed in a partnership with ISRO (Indian

Space Research Organisation).

Option (d) is incorrect: Biological

Photovoltaic Cells (BPV) offer an exciting

opportunity to use algae to harvest sunlight and generate electricity. Advantages

compared to conventional solar panels, these

are cheap, green, long lasting,

environmentally friendly, and easy to

manufacture. Tesla not ISRO is doing

research on these cells.

Page 14: ALL INDIA PRELIMS TEST SERIES - 2020...2 AIPTS 2020 (S&T - 17) (E) Answer Key Byju’s Classes: 9873643487 of the sea as methane because it is transformed into CO2. But the formation

14 AIPTS 2020 (S&T - 17) (E) Answer Key Byju’s Classes: 9873643487

58. Ans: (d) Explanation: A new 30 MHz radar system,

Gadanki Ionospheric Radar Interferometer

(GIRI), has been established at National

Atmospheric Research Laboratory (NARL). Option (a) is correct: The Indian Space

Research Organisation (ISRO) has installed

the Gadanki Ionospheric Radar

Interferometer (GIRI) Radar System near

Tirupati, Andhra Pradesh.

Option (b) is correct: It has been established for ionospheric, meteor and space weather

research in a comprehensive way.

Option (c) is correct: GIRI has been

designed with the following objectives:

to carry out unattended observations towards studying the forcing from the sun (e.g.,

variation in solar flux, solar flare, magnetic

storm) and from the underneath atmosphere

(e.g., waves generated by weather

phenomena) on the ionospheric irregularities.

Option (d) is incorrect: It has been developed by ISRO, India, but not in

collaboration with Israel.

59. Ans: (c)

Explanation: A cell is defined as the smallest, basic unit of life that is responsible for all of

life‘s processes.

Statement 1 is incorrect: Animal cells do

not have cell walls. Cell wall forms an outer

covering for the plasma membrane of fungi and plants. Algae have cell walls, made of

cellulose, galactans, mannans and minerals

like calcium carbonate, while in other plants

it consists of cellulose, hemicellulose, pectins

and proteins.

Statement 2 is incorrect: Ribosomes are the protein synthesisers of the cell. Lysosomes

protect the cell by engulfing the foreign

bodies entering the cell and helps in cell

renewal. Therefore, it is known as the cell‘s

suicide bags. Statement 3 is correct: Mitochondria are

the sites of aerobic respiration. They produce

cellular energy in the form of ATP, hence they

are called ‗power houses‘ of the cell.

60. Ans: (a) Explanation:

Option (a) is correct: Cramps are defined as

painful involuntary skeletal muscle

contractions. It can be caused-

When a muscle is not able to relax

properly (such as from a deficiency of magnesium or potassium in your diet)

When muscles become irritated by a buildup of lactic acid (which can happen

if you don't rest your muscle after it has exercised a lot).

Reduced blood flow to the muscles also can cause cramps

Option (b) is incorrect: Dopamine: It is

known as the feel-good neurotransmitter—a chemical that ferries information between

neurons.

Option (c) is incorrect: Glucose: Most of the

cells in your body use glucose along with

amino acids (the building blocks of protein)

and fats for energy, but it's the main source of fuel for your brain.

Option (d) is incorrect: Serotonin: It is a

neurotransmitter known to play a key role in

a number of functions ranging from sleep to

social behaviour. Recently, Indian scientists have discovered that serotonin boosts energy

production in brain cells and helps them

survive under stress. This new knowledge can

potentially be used to develop anti-stress

drugs in future.

61. Ans: (a)

Explanation:

Statement 1 is correct: Irradiation is the

method of using radioactive waves to kill harmful pathogens and control of spoilages in

food materials. It increases the shelf life of

the food materials like meat, vegetables and

spices. It is similar to pasteurization of milk

but the source of the energy used differs in

irradiation. Statement 2 is incorrect: Irradiation will not

have any impact on the nature of the food

materials treated. The consumers can not be

able to find any distinct odor, flavour, taste or

feel in the irradiated food materials.

Statement 3 is incorrect: Irradiation of food materials in India is not banned, but

governed by the Atomic Energy (Radiation

Processing of Food and Allied Products) Rules

2012. The Bhabha Atomic Research Centre

operates a Food Irradiation Processing Laboratory for the development of the

required technologies and its

commercialisation.

62. Ans: (c) Explanation: ITER is one of the most

ambitious energy projects in the world today.

It aims to create a reactor that will be a

carbon-free source of energy .The ITER will be

the first fusion device to produce net energy. ITER will be the first fusion device to

maintain fusion for long periods of time.

Statement 1 is incorrect: ITER uses the

vessel called tokamak to sustain the fusion

Page 15: ALL INDIA PRELIMS TEST SERIES - 2020...2 AIPTS 2020 (S&T - 17) (E) Answer Key Byju’s Classes: 9873643487 of the sea as methane because it is transformed into CO2. But the formation

15 AIPTS 2020 (S&T - 17) (E) Answer Key Byju’s Classes: 9873643487

reaction (not fission) for a longer time period. The tokamak comes from Russian acronym

that stands for toroidal chamber with

magnetic coils. It is a magnetic fusion device

that has been designed to prove the feasibility

of fusion at a large-scale. Statement 2 is correct: The main fuels used

in nuclear fusion are deuterium and tritium,

both are heavy isotopes of hydrogen.

Deuterium constitutes a tiny fraction of

natural hydrogen, only 0,0153%, and can be

extracted inexpensively from seawater. Tritium can be made from lithium, which is

also abundant in nature.

Statement 3 is correct: ITER was first

established in 1985. The ITER Members are—

China, the European Union, India, Japan, Korea, Russia and the United States.

European Union being the host party

contributes 45% of the budget cost while the

rest of the parties contribute 9% each. Hence,

India is the only country from South Asia to

be a member of the ITER research project.

63. Ans: (b)

Explanation: Robotic Process Automation is

the technology that allows anyone today to configure computer software, or a ―robot‖ to

emulate and integrate the actions of a human

interacting within digital systems to execute

any process. It provides best quality customer

service.

Pair 1 is correctly matched: Probots are bots that follow simple, repeatable rules to

process data. They are the basic bots that

follow simple rules to process information

Pair 2 is incorrectly matched: Knowbots are

used to find out and collect and save the information from the web. They are not used

to feed personal information into them and

retrieve it later.

Pair 3 is correctly matched: Chatbots are

virtual agents who can respond to customer

queries in real time.

64. Ans: (c)

Explanation:

Statement 1 is correct: Helium hydride ion

(HeH+) was the first molecule formed when, almost 14 billion years ago, falling

temperatures in the young universe allowed

recombination of the light elements produced

in the Big Bang. Helium atoms combined with

hydrogen protons to form helium hydride ion HeH+, the universe‘s first molecular bond.

Statement 2 is correct: Scientists

discovered Helium Hydride signature for the

first time in our own galaxy using the world‘s

largest airborne observatory, NASA‘s

Stratospheric Observatory for Infrared

Astronomy, or SOFIA, as the aircraft flew high above the Earth‘s surface and pointed

its sensitive instruments out into the cosmos.

65. Ans: (b) Explanation:

Statement 1 and 2 are

correct: Thermosetting plastics are plastics

which when moulded once, can not be

softened by heating. These plastics are

polymers that irreversibly become rigid when heated. They have high melting points and

tensile strength. Bakelite and melamine are

examples of thermosetting plastics.

Statement 3 and 4 are incorrect: Plastic

which gets deformed easily on heating and can be bent easily are known as

thermoplastics. Polythene and PVC are

examples of thermoplastics. These are used

for manufacturing toys, combs and various

types of containers.

66. Ans: (b)

Explanation: The transmission of audio and

video signals in optical fibres is possible

because of the phenomenon of Total Internal

Reflection. Total internal reflection is a complete reflection of a ray of light within a

medium such as optical fibre from the

surrounding surfaces back into the medium.

The phenomenon occurs if the angle of

incidence is greater than a certain limiting angle, called the critical angle.

67. Ans: (b)

Explanation:

Option (a) is incorrect: Washing soda is a chemical compound used to remove stubborn

stains from laundry. It is a salt of carbonic

acid. It is not used in fire extinguishers.

Option (b) is correct: Baking Soda is also

known as Sodium hydrogencarbonate. It decomposes on heating to generate bubbles of

carbon dioxide (leaving holes in cakes or

pastries and making them light and fluffy). It

is used as a mild antiseptic for skin

infections. It is used in fire extinguishers.

Option (c) is incorrect: Caustic soda is one of the common names for sodium hydroxide

(NaOH). Its common name derives from its

chemical identity as a sodium hydrate and

because it is caustic or corrosive, it is not

used in fire extinguishers. Option (d) is incorrect: Borax or sodium

borate, is an alkaline mineral salt with a

powdery white appearance. It is used as a

cleaning product. It is also used as Fungicide,

Herbicide and Insect killer, particularly in

Page 16: ALL INDIA PRELIMS TEST SERIES - 2020...2 AIPTS 2020 (S&T - 17) (E) Answer Key Byju’s Classes: 9873643487 of the sea as methane because it is transformed into CO2. But the formation

16 AIPTS 2020 (S&T - 17) (E) Answer Key Byju’s Classes: 9873643487

roach killing products. It is not used in fire extinguishers.

68. Ans: (a)

Explanation: Option (a) is correct: Speech 2 Face model is

MIT‘s Deep Neural Network that reconstructs

faces using only voice. It has trained a deep

neural network — a type of multilayered

artificial neural network that mimics the non-

linear architecture of the human brain — using millions of Internet videos featuring

over 100,000 talking heads. It is from these

videos that the Speech2Face AI is able to

―learn‖ the correlations between someone‘s

facial features and the sounds these features will most likely produce.

Option (b) is incorrect: Deep face

recognition model is used for imperfect face

images that often come from CCTV cameras.

In this model we explore the question that

surrounds the idea of face recognition using partial facial data.

Option (c) is incorrect: Speech2Face has

been built using VGG-Face, an existing face

recognition model, that has been developed

by University of oxford, that has been pre-trained on a large dataset of faces. It does not

do voice to face recognition.

Option (d) is incorrect: Dlib is a toolkit for

making real world machine learning and data

analysis applications. It is majorly used for

face detection and facial landmark detection. It does not include speech 2 face recognition.

69. Ans: (d)

Explanations:

Statement 1 is correct: Tuberculosis (TB) is an infectious disease caused by

Mycobacterium tuberculosis. Tuberculosis

commonly affects the lungs, but can also

affect other parts of the body.

Statement 2 is correct: It spreads from person to person through the air, when

people who are infected with TB infection

cough, sneeze or otherwise transmit

respiratory fluids through the air. Hence, it is

a communicable disease.

Statement 3 is correct: India has launched Nikshay poshan yojana for TB patients.

Under the scheme, for encouraging good

nutrition during treatment period financial

incentive of Rs.500/- per month is provided

as a nutritional support to each notified TB patient for duration for which the patient is

on anti-TB treatment. Incentives are delivered

through direct benefit transfer (DBT) scheme

to bank accounts of beneficiaries.

70. Ans: (c) Explanations: Biofortification refers to

breeding crops with higher levels of vitamins

and minerals, or higher protein and healthier

fats – is the most practical means to improve public health. Breeding for improved

nutritional quality is undertaken with the

objectives of improving –

Protein content and quality;

Oil content and quality;

Vitamin content;

Micro-nutrient and mineral content.

71. Ans: (a)

Explanation: Prototype Fast Breeder Reactor (PFBR) is a new generation of reactor, which

is capable of producing more fissile material

or fuel than what it consumes. Bharatiya

Nabhikiya Vidyut Nigam (Bhavini), a public

sector company under DAE, has been given the responsibility to build the fast breeder

reactors in India.

Statement 1 is incorrect: The fast breeder

reactor uses fast neutrons to generate more

nuclear fuels (not liquid sodium) than they

consume while generating power, dramatically enhancing the efficiency of the

use of resources. Liquid sodium is used as a

coolant in fast breeder reactors on account of

its excellent heat transfer properties

Statement 2 is correct: Under appropriate operating conditions, the neutrons given off

by fission reactions can "breed" more fuel

from otherwise non-fissionable isotopes. The

most common breeding reaction is that of

plutonium-239 from non-fissionable

uranium-238. The term "fast breeder" refers to the types of configurations which can

actually produce more fissionable fuel than

they use. This scenario is possible because

the non-fissionable uranium-238 is 140 times

more abundant than the fissionable U-235 and can be efficiently converted into Pu-239

by the neutrons from a fission chain reaction.

Statement 3 is correct: The indigenously

designed industrial scale prototype FBR of

500 MWe capacity is in an advanced stage of

commissioning at Kalpakkam. The development of FBR is part of India‘s three

stage nuclear programme devised by indian

government in 1954. The Kalpakkam PFBR is

using Uranium-238 not thorium, to breed

new fissile material, in a sodium-cooled fast reactor design.

Page 17: ALL INDIA PRELIMS TEST SERIES - 2020...2 AIPTS 2020 (S&T - 17) (E) Answer Key Byju’s Classes: 9873643487 of the sea as methane because it is transformed into CO2. But the formation

17 AIPTS 2020 (S&T - 17) (E) Answer Key Byju’s Classes: 9873643487

72. Ans: (c) Explanation:

Pair 1 is incorrectly matched: Parth is the

world‘s cheapest gun shot locator developed

by Indian Army with a private firm. It can locate the exact location of the bullet from a

distance of 400 meters which will help to

locate and neutralise terrorists faster. It

doesn't have engines powered by Indian bio

jet fuel.

Pair 2 is correctly matched: The Sagarika/K-15 missile is the SLBM

(Submarine Launched Ballistic Missile)

version of the land-based Shaurya missile.

Medium range K-15 ballistic missile has a

range between 700km to 1,500 km with varying payload.

Pair 3 is incorrectly matched: AN32 is not a

gunshot locator, but an aircraft of Indian Air

Force, whose engines are powered by Indian

bio jet fuel. The fuel is developed from non

edible ‗Tree Borne Oils‘. It will reduce carbon footprint and crude oil imports dependency.It

was developed by CSIR-IIP.

Pair 4 is correctly matched: Tejas is India‘s

Light Combat Aircraft (LCA) together with its

variants, is the smallest and lightest Multi-Role Supersonic Fighter Aircraft of its class

developed by Aeronautical Development

Agency (ADA) and Hindustan Aeronautics

Limited (HAL).

73. Ans: (a) Explanation: The Government has recently

set up a National Startup Advisory Council to

advise the Centre on measures needed to

build a strong ecosystem for nurturing

innovation and start-ups in the country. Statement 1 is correct: The Council will be

chaired by the Minister for Commerce &

Industry. Joint secretary of Department for

Promotion of Industry and Internal Trade will

be the convener of the body. Statement 2 is incorrect: The National

Startup Advisory Council will advise the

Government on measures needed to build a

strong ecosystem for nurturing innovation

and startups in the country to drive

sustainable economic growth and generate large scale employment opportunities.

Startup Incubators (not the National Start-up

Advisory Council) advises Startups on

funding, regulatory compliance, Intellectual

property rights among other things.

74. Ans: (c)

Explanation: The Forest Advisory Committee

has approved a scheme ‗Green Credit

Scheme‘ that could allow ―forests‖ to be traded as a commodity.

Statement 1 is correct: It will allow private

agencies like private companies, village forest

communities to create tree plantations on

private / non-forest land, to be used as Compensatory Afforestation for projects

involving forest diversion.

Statement 2 is correct: ‗Green Credit

Scheme‘ would allow agencies like private

companies and village forest communities to

identify land and grow plantations. After a period of three years, they would be eligible to

be considered as compensatory forest land if

they met the criteria set by the Forest

Department. An industry in need of forest

land could then pay for these patches of forest land, and this would then be

transferred to the Forest Department.

75. Ans: (b)

Explanation: Kalapani is a 372-sq km area at the China-Nepal-India tri-junction. India

claims Kalapani as a part of Uttarakhand

while Nepal depicts the area in its map. The

border dispute flared up again recently after

India released its new political map, following the reorganisation of J&K, showing the area

as its own.

According to the Sugauli treaty signed by

Nepal and British India in 1816, the Mahakali

river that runs through the Kalapani area is

the boundary between the two countries. Strategically, Lipulekh Pass in Kalapani

serves as an important vantage point for

India to keep an eye on Chinese movements.

Statement 1 is correct: With the recent

release of political map of India, after the bifurcation of the state of Jammu and

Kashmir (J&K) into the Union Territories of

J&K and Ladakh, the Kalapani

territory became the bone of contention

between India and Nepal.

Statement 2 is incorrect: Kalapani is located at an altitude of 3600m on the

Kailash Manasarovar route. It borders

Uttarakhand in India and Sudurpaschim

Pradesh in Nepal. It is not part of Uttar

Pradesh.

76. Ans: (a)

Explanation: In quantum computing,

quantum supremacy is the goal of

demonstrating that a programmable quantum device can solve a problem that classical

computers practically cannot (irrespective of

the usefulness of the problem). Though

classical computers can solve the problems, it

will take them more than 1000 years to

Page 18: ALL INDIA PRELIMS TEST SERIES - 2020...2 AIPTS 2020 (S&T - 17) (E) Answer Key Byju’s Classes: 9873643487 of the sea as methane because it is transformed into CO2. But the formation

18 AIPTS 2020 (S&T - 17) (E) Answer Key Byju’s Classes: 9873643487

complete it. so it is referred to classical computers practically cannot solve it.

Quantum computers work differently from

the classical computers we work on today.

Exploiting the principles of quantum

mechanics, they can easily tackle computational problems that may be tough

for the classical computer as the size of the

numbers and number of inputs involved

grows bigger. Quantum computers do not

look like desktops or laptops that we

associate the word ‗computer‘ with. Instead (and there are only a handful of them) they

resemble the air-conditioned server rooms of

many offices or the stacks of central

processing units from desktops of yore that

are connected by ungainly tangled wires and heaped in freezing rooms. Quantum

supremacy refers to quantum computers

being able to solve a problem that a classical

computer practically cannot.

77. Ans: (d) Explanation:

Statement 1 is correct: Chandrayaan-2 is

an Indian lunar mission that will boldly go

where no country has ever gone before the Moon's south polar region. The Vikram lander

was designed to execute the mission‘s soft

landing on the lunar surface.

Statement 2 is correct: Chandrayaan-2

orbiter utilizes X-rays emitted by the Sun in a

clever way to study elements on the lunar surface. Solar X-rays excite atoms of

constituent elements on the lunar surface.

These atoms when de-excited emit their

characteristic X-rays (a fingerprint of each

atom). By detecting these characteristic X-rays, it becomes possible to identify various

major elements of the lunar surface.

Statement 3 is correct: Moon provides the

best linkage to Earth‘s early history. It offers

an undisturbed historical record of the inner

Solar system environment. The Lunar South pole is especially interesting because the

lunar surface area that remains in shadow is

much larger than that at the North Pole.

There could be a possibility of presence of

water in permanently shadowed areas around it. In addition, the South Pole region has

craters that are cold traps and contain a

fossil record of the early Solar System.

78. Ans: (c) Explanation: SWAYAM (Study Webs of

Active–Learning for Young Aspiring Minds) is

a programme initiated by Government of

India and designed to achieve the three

cardinal principles of Education Policy viz.,

access, equity and quality. SWAYAM seeks to

bridge the digital divide for students who have hitherto remained untouched by the

digital revolution and have not been able to

join the mainstream of the knowledge

economy.

Statement 1 is incorrect: SWAYAM platform is developed by the Ministry of Human

Resource Development (MHRD) and NPTEL,

IIT Madras with the help of Google Inc. and

Persistent Systems Ltd. It would be ultimately

capable of hosting 2000 courses and 80000

hours of learning: covering school, under-graduate, post-graduate, engineering, law

and other professional courses.

Statement 2 is incorrect: Courses delivered

through SWAYAM are available free of cost to

the learners, however learners wanting a SWAYAM certificate should register for the

final proctored exams that come at a fee and

attend in-person at designated centres on

specified dates. Hence, the SWAYAM course

is free of cost. Fees are only levied if the

learner requires a certificate. Statement 3 is correct: The SWAYAM

platform facilitates hosting of all the courses,

taught in classrooms from Class 9 till post-

graduation to be accessed by anyone,

anywhere at any time. All the courses are interactive, prepared by the best teachers in

the country and are available, free of cost to

any learner. More than 1,000 specially

chosen faculty and teachers from across the

country have participated in preparing these

courses.

79. Ans: (d)

Explanation: Vigyan Samagam is a multi-

venue mega-science exhibition that brings several collaborative mega-science projects

under one roof.

Statement 1 is incorrect: It is a national

level exhibition. Vigyan Samagam is the first

ever mega science exhibition in India. Vigyan

Samagam exhibition is showcasing India‘s contribution to international collaborations

on fundamental science and research and

providing a common interactive platform for

mega-science projects, industry, academia

and institutions. Statement 2 is incorrect: Vigyan Samagam

does not aim to secure funds to promote

scientific and industrial research in India. To

secure and manage funds and endowments

for the promotion and management of science

is one of the several objectives of Indian Science Congress.

Statement 3 is incorrect: It is not part of

the 100th birth anniversary celebration of

Vikram Sarabhai. It is an exhibition to usher

Page 19: ALL INDIA PRELIMS TEST SERIES - 2020...2 AIPTS 2020 (S&T - 17) (E) Answer Key Byju’s Classes: 9873643487 of the sea as methane because it is transformed into CO2. But the formation

19 AIPTS 2020 (S&T - 17) (E) Answer Key Byju’s Classes: 9873643487

fundamental science and research as a strong career option for youngsters.

80. Ans: (c)

Explanation: INDIGO is an initiative to set up advanced experimental facilities for a

multi-institutional observatory project in

gravitational-wave astronomy located near

Aundha Nagnath, Hingoli District,

Maharashtra.

Statement 1 is incorrect: INDIGO or IndIGO (Indian Initiative in Gravitational-wave

Observations) is a consortium of Indian

gravitational-wave physicists. The three lead

institutions in the IndIGO consortium are:

Institute of Plasma Research (IPR), Inter-University Centre for Astronomy and

Astrophysics (IUCAA), and Raja Ramanna

Centre for Advanced Technology

(RRCAT). IndIGO has currently over 70

member scientists.

Statement 2 is correct: Since 2009, the IndIGO Consortium has been planning a

roadmap for gravitational-wave astronomy

and a phased strategy towards Indian

participation in realizing a gravitational-wave

observatory in the Asia-Pacific region. IndIGO is the Indian partner (along with the LIGO

Laboratory in US) in planning the LIGO-India

project. LIGO-India is a planned advanced

gravitational-wave detector to be located in

India, whose concept proposal is now under

active consideration by the science funding agencies in India and US.

Statement 3 is correct: IndiGO is a

member of the LIGO Scientific Collaboration.

The LIGO Scientific Collaboration (LSC) is a

scientific collaboration of international physics institutes and research groups

dedicated to the search for gravitational

waves.

81. Ans: (c) Explanations: Neglected tropical diseases

(NTDs) are a diverse group of communicable

diseases that prevail in tropical and

subtropical conditions in 149 countries. It

affects more than one billion people and costs

developing economies billions of dollars every year. Populations living in poverty, without

adequate sanitation and in close contact with

infectious vectors and domestic animals and

livestock are those worst affected. Some of the

neglected disease are Buruli ulcer, Chagas disease, Dengue and Chikungunya,

Dracunculiasis (guinea-worm disease),

Echinococcosis, Foodborne trematodiases,

Human African trypanosomiasis (sleeping

sickness), Leishmaniasis, Leprosy (Hansen's

disease), Lymphatic filariasis, Mycetoma,

chromoblastomycosis and other deep mycoses, Onchocerciasis (river blindness),

Rabies, Scabies and other ectoparasites,

Schistosomiasis, Soil-transmitted

helminthiases, Snakebite envenoming,

Taeniasis/Cysticercosis, Trachoma and Yaws (Endemic treponematoses).

Encephalitis has not been included in the list

of Neglected tropical diseases.

82. Ans: (d) Explanation: Nobel Prize in Physiology or

Medicine for 2019 was awarded to William G

Kaelin, Jr, Peter J Ratcliffe and Gregg L

Semenza.They received the award jointly for

their discoveries of "how cells sense and adapt to oxygen availability.‖ This discovery

has paved the way for promising new

strategies to fight anemia, cancer and many

other diseases. Their research established the

basis for the understanding of how oxygen

levels affect cellular metabolism and physiological functions.

Statement 1 is incorrect: The research of

Nobel Prize winners revealed that the oxygen

sensing mechanism is present in virtually all

tissues, and not only in the kidney cells. While the kidney releases Erythropoietin, the

rest of the cells adapt to hypoxia through a

protein complex named ‗hypoxia-inducible

factor‘ (HIF).

Statement 2 is correct: Oxygen travels in

our body through the Red Blood Cells(RBCs) to the different tissues. The mitochondria in

the cells then use oxygen to convert food into

energy. In situations, where there‘s a lack of

oxygen, our body suffers from hypoxia, a

condition where tissues are unable to perform bodily functions due to a lack of oxygen. Our

body‘s automatic response to hypoxia is to

increase the levels of the Erythropoietin

(EPO), a hormone produced by the kidneys.

Statement 3 is correct: Erythropoiesis is the

process which produces red blood cells, which is the development from erythropoietic

stem cell to mature red blood cell. It is

stimulated by decreased Oxygen in

circulation, which is detected by the kidneys,

which then secrete the hormone erythropoietin.

83. Ans: (d)

Explanation:

Statement 1 is correct: Muntra is India's first unmanned tank and is an acronym for

Mission Unmanned TRAcked. It was rolled by

DRDO in 2017.

Statement 2 is incorrect: Autonomous

Underwater Vehicle AUV-150 can be deployed

for seabed mapping, coastal surveillance,

Page 20: ALL INDIA PRELIMS TEST SERIES - 2020...2 AIPTS 2020 (S&T - 17) (E) Answer Key Byju’s Classes: 9873643487 of the sea as methane because it is transformed into CO2. But the formation

20 AIPTS 2020 (S&T - 17) (E) Answer Key Byju’s Classes: 9873643487

oceanographic measurements during adverse weather conditions. Muntra cannot be

deployed for seabed mapping and coastal

surveillance.

Statement 3 is correct: There are three

variants of Muntra and they are deployed for unmanned surveillance (Muntra S), detecting

mines (Muntra M) and for reconnaissance in

areas with risk of nuclear radiation or bio

weapon (Muntra N).

84. Ans: (c) Explanation:

Statement 1 is correct: Stem cells are

special human cells that can develop into

many different types of cells, from muscle cells to brain cells. Stem cells are

undifferentiated cells of multicellular

organisms which are capable of giving rise to

indefinitely more cells (through mitosis) of the

same type and from which certain other

kinds of cell may be formed by the cellular differentiation. In some cases, they also have

the ability to repair damaged tissues.

Statement 2 is incorrect: Stem cells can be

derived from both mammals and non-

mammals. Induced pluripotent stem cells are derived from adult or mature cells.

Statement 3 is correct: Embryonic stem

cells come from human embryos that are

three to five days old. They are harvested

during a process called in-vitro fertilization.

They are known as pluripotent stem cells. These cells can give rise to virtually any other

type of cell in the body.

85. Ans: (c)

Explanation: The Global Mobility Report is the first-ever study released by Sustainable

Mobility for All (SuM4All) 2017 to assess the

global performance of the transport sector

and the progress made toward four main

objectives: universal access, efficiency, safety, and green mobility. The publication covers all

modes of transport, including road, air,

waterborne, and rail transport. According to

the report, the world is not on track to

achieving sustainable mobility. It states that

the transport sector, apart from being inaccessible to many of the world‘s most

vulnerable, is plagued by high fossil fuel use,

rising greenhouse gas emissions, air and

noise pollution, an alarming number of road

fatalities and a reluctance to embrace digitalization.

Additional Information about Sustainable

Mobility for All

In 2016, the World Bank proposed bringing

unity to the mobility sector by bringing

interested stakeholders under one umbrella, to support the implementation of the

Sustainable Development Goals and achieve

sustainable mobility. Out of this call for

action, Sustainable Mobility for All was born.

In January 2017, the initiative was formally established during the first Consortium

Meeting in Washington, DC. Sustainable

Mobility for All brings together a diverse and

influential group of transport stakeholders,

with a commitment to speak with one

coherent voice and act collectively to implement the Sustainable Development

Goals (SDG‘s) and transform the transport

sector.

86. Ans: (a) Explanation: WHO declared the outbreak of

nCoV as a Public Health Emergency of

International Concern.

Statement 1 is correct: Coronaviruses (CoV)

are a large family of viruses that cause illness ranging from the common cold to more severe

diseases such as Middle East Respiratory

Syndrome (MERS-CoV) and Severe Acute

Respiratory Syndrome (SARS-CoV). A novel

coronavirus (nCoV) is a new strain that has not been previously identified in humans.

Statement 2 is incorrect: The W.H.O. says

the global death rate of the novel coronavirus

is 3.4 percent. The Middle East Respiratory

Syndrome has a fatality rate of 34.4%

globally. So, currently nCoV clearly does not have the highest mortality rate among all

viral diseases.

87. Ans: (c)

Explanation: Statement 1 is correct: Gaganyaan is

India's first manned space mission which the

ISRO aims to launch by December 2021.

Gaganyaan will be the Indian crewed orbital

spacecraft intended to be the basis of Indian Human Space Flight Program. The target is to

launch it before the 75 year celebration of

India‘s independence.

Statement 2 is incorrect: GSLV Mk III, the

three-stage heavy lift launch vehicle, will be

used to launch Gaganyaan as it has the necessary payload capability. The mission will

have a payload of 7 ton. It will be launched by

India‘s biggest rocket GSLV Mk 3 from

Sriharikotta.

Statement 3 is correct: It will circle Earth at a low-earth-orbit at an altitude of 300-400

km from earth for 5-7 days. Technically,

objects in low-Earth orbit are at an altitude of

between 160 to 2,000 km above the Earth‘s

surface. The mission will be placed in the Low

Page 21: ALL INDIA PRELIMS TEST SERIES - 2020...2 AIPTS 2020 (S&T - 17) (E) Answer Key Byju’s Classes: 9873643487 of the sea as methane because it is transformed into CO2. But the formation

21 AIPTS 2020 (S&T - 17) (E) Answer Key Byju’s Classes: 9873643487

Earth orbit. The International Space Station (ISS) is also placed in this orbit.

Statement 4 is correct: With the successful

launch of Gaganyaan, India will become only

the 4th country after Russia, the USA and

China to send humans to space.

88. Ans: (b)

Explanation:

Option (a) is correct: If the surface area is

increased, the rate of evaporation increases. For example, while putting clothes for drying

up we spread them out. Evaporation is a

surface Phenomenon.

Option (b) is incorrect: Humidity is the

amount of water vapour present in air. The air around us cannot hold more than a

definite amount of water vapour at a given

temperature. If the amount of water vapour in

air is already high, the rate of evaporation

decreases.

Option (c) is correct: It is a common observation that clothes dry faster on a windy

day. With the increase in wind speed, the

particles of water vapour move away with the

wind, decreasing the amount of water vapour

in the surrounding. Option (d) is correct: With the increase of

temperature, more particles get enough

kinetic energy to go into the vapour state.

89. Ans: (a) Explanation:

Statement 1 is correct: Anodising is a

process of forming a thick oxide layer of

aluminium. Aluminium develops a thin oxide

layer when exposed to air. This aluminium

oxide coat makes it resistant to further corrosion. The resistance

can be improved further by making the oxide

layer thicker.

Statement 2 is incorrect: Aqua regia, is a

freshly prepared mixture of concentrated hydrochloric acid and concentrated nitric

acid (not sulphuric acid) in the ratio of 3:1. It

can dissolve gold, even though neither of

these acids can do so alone. Aqua regia is a

highly corrosive, fuming liquid. It is one of the

few reagents that is able to dissolve gold and platinum.

90. Ans: (a)

Explanation: The Miyawaki technique was pioneered by Japanese botanist Akira

Miyawaki. The Miyawaki method of

afforestation has revolutionised the concept

of urban afforestation. The Kerala

government is replicating the model across

Kerala, which has suffered floods and soil

erosion, as a part of the Rebuild Kerala initiative. This process will involve turning

backyards into mini-forests and it is to come

upon the government office premises,

residential complexes, school premises, and

puramboke land in Kerala state.

91. Ans: (b)

Explanation: Reang (aka Bru in Tripura and

Mizoram) are one of the 21 scheduled tribes

of the Indian state of Tripura. The Bru can be found all over the Tripura state in India.

However, they may also be found in Mizoram

and Assam. They speak the Reang dialect of

Bru language which is of Tibeto-Burmese

origin and is locally referred to as Kau Bru.

Statement 1 is incorrect: The recent

agreement related to ―Bru refugees is a four

party (or quadripartite) agreement among the

Centre, Mizoram Government, Tripura

Government and Mizoram Bru Displaced People‘s Forum (MBDPF) which sought to end

the 22-year-old Bru refugee crisis. This

agreement will bring a permanent solution for

the rehabilitation of thousands of Bru-Reang

people in Tripura. Statement 2 is correct: Union Home

Minister presided over the signing of a

quadripartite pact of MHA with the State

governments of Tripura, Mizoram and leaders

of Bru community to permanently settle

around 34,000 internally displaced people in Tripura.

92. Ans: (c)

Explanation:

Statement 1 is correct: In order to utilise and harness the benefits of Cloud

Computing, Government of India has

embarked upon an ambitious initiative - "GI

Cloud" which has been named as 'MeghRaj'.

This initiative is to implement various components including governance

mechanism to ensure proliferation of Cloud in

the government.

Statement 2 is correct: Meghdoot is a

mobile app launched to assist farmers. It has

been developed by the Indian Meteorological Department and Indian Institute of Tropical

meteorology and the Indian Council of

Agricultural Research. The app provides

forecasts relating to temperature, rainfall,

humidity, and wind speed and direction which play critical roles in agricultural

operations and advisories to the farmers on

how to take care of their crops and livestock.

Page 22: ALL INDIA PRELIMS TEST SERIES - 2020...2 AIPTS 2020 (S&T - 17) (E) Answer Key Byju’s Classes: 9873643487 of the sea as methane because it is transformed into CO2. But the formation

22 AIPTS 2020 (S&T - 17) (E) Answer Key Byju’s Classes: 9873643487

93. Ans: (b) Explanation: Zero-day vulnerability is a

computer-software vulnerability that is

unknown to, or unaddressed by, those who

should be interested in mitigating the vulnerability. Until the vulnerability is

mitigated, hackers can exploit it to adversely

affect computer programs, data, additional

computers or a network. An exploit directed

at a zero-day is called a zero-day exploit, or

zero-day attack.

94. Ans: (b)

Explanation: The Reserve Bank of India (RBI)

has chalked out an ambitious strategy for financial inclusion to create the necessary

infrastructure to move towards a less-cash

society by March 2022.

Statement 1 is incorrect: The strategy was

launched by the RBI (not NITI Aayog) for the

period 2019-2024. Statement 2 is correct: The strategy aims to

strengthen the ecosystem for various modes

of digital financial services in all Tier-II to Tier

VI centres to create the necessary

infrastructure to move towards a less-cash

society by March 2022. Statement 3 is correct: Under the strategy,

there is a plan to make the Public Credit

Registry (PCR) fully operational by March

2022 so that authorised financial entities

could leverage the same for assessing credit proposals from all citizens.

95. Ans: (c)

Explanations: Helminths (worms) which are

transmitted through soil contaminated with faecal matter are called soil-transmitted

helminths (Intestinal parasitic worms).

Statement 1 is correct: Roundworm (Ascaris

lumbricoides), whipworm (Trichuris trichiura)

and hookworms (Necator americanus and Ancylostoma duodenale) are worms that

infect people with Soil Transmitted

Helminths.

Statement 2 is incorrect: Soil-transmitted

helminths are transmitted by eggs that are

passed in the faeces of infected people. Adult worms live in the intestine where they

produce thousands of eggs each day. In areas

that lack adequate sanitation, these eggs

contaminate the soil. There is no direct

person-to-person transmission, or infection from fresh faeces, because eggs passed in

faeces need about 3 weeks to mature in the

soil before they become infectious.

Statement 3 is correct: To eradicate STH,

Indian government has launched a National

Deworming programme. It aims at eradicating

intestinal worms also known as Soil-Transmitted Helminths (STH), among

children in the age group of 1-19 years.

Under the programme, children and

adolescents are administered a single dose of

a safe medicine Albendazole across government, government-aided schools,

anganwadis, private schools and other

educational institutions.

96. Ans: (a) Explanation: A virtual private network, or

VPN, is an encrypted connection over the

Internet from a device to a network. The

encrypted connection helps ensure that

sensitive data is safely transmitted. It prevents unauthorized people from

eavesdropping on the traffic and allows the

user to conduct work remotely. VPN

technology is widely used in corporate

environments.

Statement 1 is correct: A VPN is a private network that uses a public network (usually

the internet) to connect remote sites or users

together. VPN technology was developed to

allow remote users and branch offices to

access corporate applications and resources. To ensure security, the private network

connection is established using an encrypted

layered tunneling protocol, and VPN users

use authentication methods, including

passwords or certificates, to gain access to

the VPN. Statement 2 is correct: The VPN uses

"virtual" connections routed through the

internet from the user's private network or a

third-party VPN service to the remote site or

person. VPNs mask the user's internet protocol (IP) address so their online actions

are virtually untraceable.

Statement 3 is incorrect: VPNs help ensure

security and privacy— anyone intercepting

the encrypted data can't read it.

97. Ans: (c)

Explanation: Mission Shakti demonstrated

the ―nation‘s capability to defend its assets in

the outer space,‖ besides vindicating the

technological competence of the Defence Research and Development Organisation

(DRDO), the exclusive research and

development (R&D) department of the

Ministry of Defence

Statement 1 is incorrect: The test was done in the Lower Earth Orbit to ensure that there

is no space debris. Whatever debris that is

generated will decay and fall back onto the

earth within weeks

Page 23: ALL INDIA PRELIMS TEST SERIES - 2020...2 AIPTS 2020 (S&T - 17) (E) Answer Key Byju’s Classes: 9873643487 of the sea as methane because it is transformed into CO2. But the formation

23 AIPTS 2020 (S&T - 17) (E) Answer Key Byju’s Classes: 9873643487

Statement 2 is correct:. There are multiple ways to demonstrate ASAT capabilities such

as "fly-by tests‖ and Jamming. India used the

particular technology of Kinetic Kill- Direct

Ascent Kinetic Kill method as it was found

appropriate to achieve the objectives set out in the mission. In direct ascent kinetic kill,

ballistic missiles are used to put an

interceptor on a ballistic trajectory that

intersects with the target in orbit without the

interceptor entering orbit itself.

Other ways to demonstrate ASAT capability without causing debris would be to do a fly-

by test, where the ground-based direct ascent

missile will fly by the targeted satellite

without destroying it. Satellites can also be

sabotaged by jamming satellites using space-based lasers. These methods fall under the

category of ―soft-kill‖ methods and do not

create debris.

Statement 3 is correct: India emerged as

the fourth member of the exclusive club of

space powers (US, Russia and China) that have demonstrated ASAT capabilities.

98. Ans: (d)

Explanation: The latest Levels and Trends in Child Mortality: Report 2019 from UNICEF

and partners in the UN Inter-Agency Group

for Child Mortality Estimation (UN IGME),

shows the full scope of child mortality rates

across the world – from newborns to

adolescents – as well as the progress made toward meeting the SDG targets by 2030.

According to the report, half of all under-5

deaths in 2018 occurred in five countries:

India, Nigeria, Pakistan, the Democratic

Republic of the Congo and Ethiopia. India and Nigeria alone account for about a third of

these deaths.

Additional information

About UN Inter-Agency Group for Child

Mortality Estimation (UN IGME)

The United Nations Inter-agency Group for Child Mortality Estimation (UN IGME)

produces estimates of child and young adolescent mortality annually, reconciling the

differences across data sources and taking

into account the systematic biases associated

with the various types of data on child and

adolescent mortality. This report presents UN IGME‘s latest estimates of neonatal, infant

and under-five mortality as well as mortality

among children aged 5–14 years.

99. Ans: (b) Explanation: Recently, yellow rust has been

reported in a few villages of Ropar,

Hoshiarpur and Pathankot districts in Punjab

and villages of Panchkula, Yamunanagar and

Ambala districts in Haryana. Statement 1 is incorrect: Wheat crop is

more susceptible to Yellow rust disease. The

detection of yellow rust disease in the wheat

crop in sub-mountainous parts of Punjab and

Haryana has raised anxiety among farmers

about a drop in the crop's yield. Statement 2 is correct: Yellow rust is a

fungal disease.

Statement 3 is correct: Yellow rust turns

the crop's leaves into a yellowish colour and

stops photosynthesis activity, which eventually could result in a drop of wheat

crop productivity.

100. Ans: (d)

Explanation: Statement 1 is incorrect: Classical smog

occurs in cool humid climates. It is a mixture

of smoke, fog and sulphur dioxide.

Chemically it is a reducing mixture and so it

is also called reducing smog.

Statement 2 is incorrect: Classical smog is formed as a result of interactions between

smoke, fog and sulphur dioxide.

Photochemical smog is formed primarily as a

result of interactions among nitrogen

oxides, reactive hydrocarbons, and sunlight.